Triangles (1 Viewer)

Estel

Tutor
Joined
Nov 12, 2003
Messages
1,261
Gender
Male
HSC
2005
The quick question:
Is it possible to have a right-angled triangle where the hypotenuse is not divisible by 5 if all sides have lengths of integer values? Can't think of a counter example right now...

The long question:
Does anyone have a proof for this (if there is no counterexample)?
 
C

coca cola

Guest
counter example 21, and 20 for non-hyp sides.

hyp = 29.

umm im let me have a think about a general proof.
 
C

coca cola

Guest
well couldn't you go by using the greek's pythagorian triple: (2m)^2 + (m^2 - 1)^2 = (m^2 + 1)^2, which can be prooved to be true for all positive intergers by using induction.

then the hyp is (m^2 + 1). this is definately NOT divisible by 5 for all intergers.

i.e. m^2 + 1 can not be expressed as 5n in general.

edit: hey have you got a different proof to this?
 
Last edited by a moderator:

Estel

Tutor
Joined
Nov 12, 2003
Messages
1,261
Gender
Male
HSC
2005
No proof...
the actual question is prove that in a right triangle with sides a, b, and c with gcd(a,b,c)=1 only one of a, b and c will be divisible by 5.
 

CM_Tutor

Moderator
Moderator
Joined
Mar 11, 2004
Messages
2,644
Gender
Male
HSC
N/A
Estel said:
No proof...
the actual question is prove that in a right triangle with sides a, b, and c with gcd(a,b,c)=1 only one of a, b and c will be divisible by 5.
It's easy to show that no more than one of a, b and c will be divisible by 5. If two of them are divisible by 5, then so is the third, in which case gcd(a, b, c) ≠ 1, which is a contradiction. All 3 divisible by 5 leads trivially to the same contradiction. Thus, no more than one of them is divisible by 5 - but I don't see off hand a way to prove that one of them must be divisible by 5.
 

ngai

Member
Joined
Mar 24, 2004
Messages
223
Gender
Male
HSC
2004
well if u use that (2m)^2 + (m^2 - 1)^2 = (m^2 + 1)^2
then a bashy method:
m is integer, so m has remainder of r = 0,1,2,3,4 when divided by 5
r = 0, then 2m divisible by 5
r = 1, then m^2 - 1 is divisible by 5
r = 2, then m^2+1
r = 3, then m^2 + 1
r = 4, then m^2 - 1
and then using cm tutor's stuff
u get that theres one and only one is divisible
 

Rorix

Active Member
Joined
Jun 29, 2003
Messages
1,818
Gender
Male
HSC
2005
greatest common divisor

but im sure you could have found that on Google or something in less than a minute:p
 

mojako

Active Member
Joined
Mar 27, 2004
Messages
1,333
Gender
Male
HSC
2004
Didnt try google:p

BTW Estel, what topic does this question come under?
 

mojako

Active Member
Joined
Mar 27, 2004
Messages
1,333
Gender
Male
HSC
2004
I just don't think this is part of the HSC 3U course... :confused:
I think I've seen this somewhere but I don't think it's in year 11 or 12:
(2m)^2 + (m^2 - 1)^2 = (m^2 + 1)^2
and my memory says that it's not to be memorised when I saw it...
 

Estel

Tutor
Joined
Nov 12, 2003
Messages
1,261
Gender
Male
HSC
2005
What topic does this come under?

Interest...
or...
Geometry if you look at the syllabus long and hard enough :p

ngai
the problem with that is it ignores triads like 7 24 25
So that is not the solution :p
 

mojako

Active Member
Joined
Mar 27, 2004
Messages
1,333
Gender
Male
HSC
2004
so, did you find the question from a HSC-related textbooks/papers??
if not I wont worry about that :p
If yes, I might worry, because I know the probability of it appearing on a HSC exam paper is very low, whether it be 2U, 3U or 4U or general maths ^^
 

Estel

Tutor
Joined
Nov 12, 2003
Messages
1,261
Gender
Male
HSC
2005
Not from HSC-related textbooks/papers...
but where's your curiosity?

Expand your horizons :p
 

ngai

Member
Joined
Mar 24, 2004
Messages
223
Gender
Male
HSC
2004
Estel said:
ngai
the problem with that is it ignores triads like 7 24 25
So that is not the solution :p
grr who gave me that dodgy formula?

anyway, try this instead: (i'm gonna use modular arithmetic...)
a^2 + b^2 = c^2
now, for an integer, say x: (== means congruent)
x == 0 (mod 5) gives x^2 == 0 (mod 5)
x == 1 (mod 5) gives x^2 == 1 (mod 5)
x == 2 (mod 5) gives x^2 == 4 == -1 (mod 5)
x == 3 (mod 5) gives x^2 == 9 == -1 (mod 5)
x == 4 (mod 5) gives x^2 == 16 == 1 (mod 5)
so x^2 == 1, -1, or 0 (mod 5)
and if x^2 == 0, then x == 0

if either of a^2 or b^2 == 0 (mod 5), then we have a or b == 0 (mod 5), so we have one divisible by 5

if neither a^2 nor b^2 == 0 (mod 5):
if a^2 == 1, b^2 == 1: c^2 == 2, impossible
if a^2 == 1, b^2 == -1: c^2 == 0, hence c divisible by 5
if a^2 == -1, b^2 == -1: c^2 == -2, impossible

and for those who dunno mod arithmetic, well most simply: "A == B (mod C)" means that when A is divided by C, there is a remainder of B
 

Estel

Tutor
Joined
Nov 12, 2003
Messages
1,261
Gender
Male
HSC
2005
damn...
I really regret not doing those enrichment books :/
 

mojako

Active Member
Joined
Mar 27, 2004
Messages
1,333
Gender
Male
HSC
2004
Estel said:
Not from HSC-related textbooks/papers...
but where's your curiosity?

Expand your horizons :p
I mean, as an observer who happened to read your thread, which deals with things I haven't learnt (eg modular arithmetics), I don't really care if I can prove that or not.
 

Estel

Tutor
Joined
Nov 12, 2003
Messages
1,261
Gender
Male
HSC
2005
mojako that doesn't rule out that it doesn't need modular arithmetic tho...
it's quite possible that it doesn't... just because 5 ppl have seen it and not found a solution...

so ppl, if you see a way, do post :)
 

Slidey

But pieces of what?
Joined
Jun 12, 2004
Messages
6,600
Gender
Male
HSC
2005
stel: I do too, but all is not entirely lost. There is a set of extremely good books on the subject called "The Art of Problem Solving" volumes 1 and 2. They cover basically everything you need to know up to precalculus. That includes conics, vectors, matrices, number theory, combinatorics, complex numbers and modular arithmetic.

http://www.artofproblemsolving.com/Books/AoPS_B_About.php

Here is the index for books one and two, if you want to see exactly what they cover:
Book1: http://www.artofproblemsolving.com/Books/AoPS_B_V1.php
Book2: http://www.artofproblemsolving.com/Books/AoPS_B_V2.php
 

Archman

Member
Joined
Jul 29, 2003
Messages
337
Gender
Undisclosed
HSC
N/A
btw i think the proper formula for triads with gcd of 1 is
(p^2-q^2)^2 + (2pq)^2 = (p^2+q^2)^2

(w00t, first post in the ext1 forum)
 

Users Who Are Viewing This Thread (Users: 0, Guests: 1)

Top